User avatar
 
the_sp4de
Thanks Received: 0
Vinny Gambini
Vinny Gambini
 
Posts: 1
Joined: May 22nd, 2012
 
 
 

Diagram

by the_sp4de Thu Oct 04, 2012 4:32 pm

This game seems easy enough but I must be missing a key inference or two to alleviate some of the work. Does anyone have a diagram?

Thanks
User avatar
 
demetri.blaisdell
Thanks Received: 161
LSAT Geek
 
Posts: 198
Joined: January 26th, 2011
 
 
 

Re: Diagram

by demetri.blaisdell Fri Oct 05, 2012 5:32 pm

My diagram is below. The key to this game is to notice that if T is in 1, U (and S) are in 2. If T is in 2, Q (and P) are in 1. Other than that, there aren't a lot of up front inferences. Let me know if you have any questions.

Demetri

Image
 
matthewyoung2008
Thanks Received: 6
Vinny Gambini
Vinny Gambini
 
Posts: 13
Joined: May 16th, 2012
 
This post thanked 1 time.
 
 

Re: Diagram

by matthewyoung2008 Sat Mar 23, 2013 12:43 am

i would draw out all three frames -- and BOOYAH -- you are done

Frame 1:

K, M, S+U are floaters

QP / 1
T / 2

Frame 2:

QP / 1
SU / 2
KMT / 3

Frame 3:

T(M or K) / 1
SU / 2
PQ(K or M) / 3
 
matthewyoung2008
Thanks Received: 6
Vinny Gambini
Vinny Gambini
 
Posts: 13
Joined: May 16th, 2012
 
 
 

Re: Diagram

by matthewyoung2008 Sat Mar 23, 2013 12:46 am

sorry, forgot to mention that the way you come up with the 3 frames is that

either P or T in Zone 1 (not both)
either T or U in Zone 2 (not both)

so frame 1 is P/1 and T/2, frame 2 is P/1 and U/2, and frame 3 is T/1 and U/2

this sets up 3 frames instead of 4 since T is repeated in both rules

thanks, LSAT test-writers!
 
rccallahan73
Thanks Received: 0
Vinny Gambini
Vinny Gambini
 
Posts: 1
Joined: April 10th, 2016
 
 
 

Re: Diagram

by rccallahan73 Wed Nov 23, 2016 1:25 pm

I'm having trouble figuring out why there are only four slots for each zone in the diagram above. Since the rules don't say that each zone has to have a sales rep in it, couldn't one zone have all 7? So if any one zone can have all of the sales reps, then I am only able to reduce the slots down to 5 each based on the rules. Anybody have some insight into this?
User avatar
 
ohthatpatrick
Thanks Received: 3807
Atticus Finch
Atticus Finch
 
Posts: 4661
Joined: April 01st, 2011
 
 
 

Re: Diagram

by ohthatpatrick Mon Nov 28, 2016 2:49 pm

Since PQ and SU are stuck together, you're bound to get at least two people in zone 1 or 2.

Try avoiding it, in order to get a 1:1:5 distribution, with the 5 going in Zone 3.

If we use T for Zone 1, then we have to use US for Zone 2. So it's 1:2:4 at that point.

If we use P for Zone 1, then we have to use PQ in Zone 1. So it's 2:1:4 at that point.